LSAT and Law School Admissions Forum

Get expert LSAT preparation and law school admissions advice from PowerScore Test Preparation.

 Administrator
PowerScore Staff
  • PowerScore Staff
  • Posts: 8919
  • Joined: Feb 02, 2011
|
#40571
Complete Question Explanation
(The complete setup for this game can be found here: lsat/viewtopic.php?t=8568)

The correct answer choice is (B)

To attack this question, use proper List question technique: take one rule and apply it to all answer choices, then take another rule, and apply it to the remaining answer choices, and so on, until only one answer choice remains. However, do not just take the rules in the order given. Instead, choose the rules to apply in order of the ease of seeing them visually inside each answer choice. In this game, the rules should be applied in this order: first, second, and third (the second and third rules are identical in form, so they can be applied in either order).

Answer choice (A): This answer choice contains M and P, and therefore it violates the first rule and is incorrect.

Answer choice (B): This is the correct answer choice.

Answer choice (C): This answer choice contains S but not T, and therefore it violates the second rule and is incorrect.

Answer choice (D): This answer choice contains S but not P, and therefore it violates the second rule and is incorrect.

Answer choice (E): This answer choice contains W but not M, and therefore it violates the third rule and is incorrect.

Get the most out of your LSAT Prep Plus subscription.

Analyze and track your performance with our Testing and Analytics Package.